The question stem reads: The reasoning in the lawyer's argument is most vulnerable to criticism on the grounds that the argument… This is a Flaw question.

The lawyer begins by making an analogy. He claims that a body of circumstantial evidence is similar to a rope. He claims that each piece of evidence is like a strand in that rope: just as adding more strings to the rope makes a rope stronger, adding more pieces of evidence strengthens the body of evidence. He then describes how if a strand of a rope is broken, the rope does not break, and it still retains much of its strength. He concludes that, similarly, if you discredit ("break") a few pieces of evidence, the overall body of evidence is still strong.

When analyzing an argument that uses an analogy, a good first step is to ask yourself, "Are the two things being compared actually similar?" As you increase the points of difference between the two things being compared, the analogy's strength diminishes. In this case, we want to determine where the lawyer's analogy between ropes and bodies of evidence frays apart. The idea that adding pieces of evidence to the body increases the strength of the body, like adding strands to a rope, makes sense and seems like a pretty good point of comparison. However, the analogy fails when we consider the fact that strands of rope are all the same. However, not all pieces of evidence are equal: some add much more strength than others. You have experience with this on the LSAT. Take away a premise that strengthens the argument, and the argument can survive. Take away a premise necessary to the argument, and the argument falls apart. So if we took away a few pieces of necessary evidence, the body would fall apart. However, that is contrary to the lawyer's conclusion. If you didn't see this, that is ok! When doing POE, prioritize answer choices that draw a distinction between ropes and bodies of evidence.

Correct Answer Choice (A) is what we discussed. The lawyer takes for granted that no evidence is more important to the body than others.

Answer Choice (B) is wrong. If you picked (B), you likely had trouble determining what (B) means. (B) says to take the strength of each piece of evidence independently and add them up. That will be greater than the strength of the evidence if you take the pieces altogether. If anything, the opposite is true: adding many pieces of circumstantial evidence together tends to count as better evidence than taking each individually.

Answer Choice (C) is not a problem for the argument. If you interpret "many = few": The point of the lawyer's argument is to show that if you take away some strands of evidence, then the body retains its strength, so the possibility is addressed. If you interpret "many"> few": then sure, the possibility is ignored. However, that is not a problem for the argument because the lawyers' conclusion is limited to taking away a few pieces of evidence. Either way, the argument is not flawed because of (C).

Answer Choice (D) is tempting, but we run into problems with the word "any." The lawyer has indicated that bodies of evidence share similarities to ropes. Adding more pieces of evidence or strands increases the strength of both.

Answer Choice (E) is incorrect. The lawyer does not use his own premise as a conclusion.


21 comments

Please note that the question stem does not stipulate that the three authors are the only authors who could be included. Therefore, we must not presume that there are no other authors.

We’ve got an MBT question here, which we can tell based on the question stem: If the statements above are true, which one of the following must be true?

We’ve got a stimulus here that almost reads like a set of logic games rules. We know that we’ve got a textbook that will have essays written by some combination of Lind, Knight, and Jones, but not all three. We also know that if the book has an essay by Knight, it will also contain an essay by Jones (so K→J).

So what are our possible sets of authors: Well what do we know? KJ is definitely a possibility. We could also always have a book that only features one author ( assuming that author isn’t K because of the aforementioned rule), so we can add “J” and “L” to our list. And then of course we could have a combination of J and L. So all the possible options of authors should look like this:
J
L
KJ
JL

This synthesizes all the rules we’ve been given, so let’s move on to the answer choices:

Correct Answer Choice (A) This fits with what we’ve determined. There is only one possible set that contains Knight: KJ. Therefore, this is correct.

Answer Choice (B) This is incorrect. We have two possible sets that contain two authors.

Answer Choice (C) This is incorrect. We have one possible combination in our list that includes Knight.

Answer Choice (D) This is incorrect. It is possible for Lind to appear on their own (i.e. without Jones).

Answer Choice (E) This is incorrect. We have two possible sets that do not include Lind.

 


1 comment

We’ve got an MSS question here which we can identify from the question stem: The statements above, if true, most strongly support which one of the following?

Our author starts with a sentence that is way too long: they tell us that a certain group of people (who are opposed to a proposed waste storage site based on extremely implausible scenarios in which the proposed site fails to contain waste) are overlooking the massive risks that would arise from waiting to move the waste from its current site.

You might notice that I put some of that information in parentheses. This is a tool that writers use in normal writing to make sentences more easily digestible and understandable. We separate bits of modifying information and context away from the core sentence so that readers can better understand the meaning of the text. The LSAT doesn’t do that here because the goal is to confuse you. Nonetheless, the information that I’ve offset with parentheses is an extremely long phrase that is modifying the word “people.” We have a set of people who object to a new waste storage site based on an implausible scenario in which the site fails to contain the waste. What these people fail to realize is that there’s a risk to just leaving this waste where it is.

In the next sentence we’re told that if we wait to move the waste until we find the perfect site, it’s going to stay where it is for the foreseeable future because it’s impossible to guarantee that there’s a site that meets all the necessary criterion. The author then explains that leaving the waste where it is “for that long” creates “unacceptable risks.”

It’s easy to get lost in the big picture of this stimulus, but if we break it down into bite sized chunks, it’s much more manageable. We’ve worked through the stimulus methodically, so hopefully we all have a good grasp of the information as we move into the answer choices:

Answer Choice (A) We don’t have any information about where the waste should have been stored in the past. We only have information about the relative risks of keeping the waste in its current location at this moment in time.

Answer Choice (B) This runs contrary to what our stimulus tells us in the second sentence. We’re told that it’s unclear that we’ll find a site that matches all of our criteria for an ideal site. The stimulus is arguing that we need to commit to the current proposal immediately instead of waiting to find the best possible option.

Correct Answer Choice (C) The author has argued that the current proposal is much better than nothing and has claimed that there are substantial risks that stem from leaving the waste in its current location. Our stimulus supports the notion that there is greater risk presented by the current site than by the proposed new site, so this answer choice is supported.

Answer Choice (D) This is too broad. We know that in this specific case time is of the essence because there are risks associated with waiting to find a better site. We don’t have any information to support the idea that this is a broad principle that should be applied every time people are locating alternative waste sites. What if the waste is in a relatively good location with moderate risk and all the alternatives present much higher risks? That would be a hypothetical in which it would be advantageous to wait until we find a better site.

Answer Choice (E) Again, this is too broad. The stimulus clearly argues for one specific site. We don’t have anything that supports the idea that any site is better than the current site.


24 comments

We’ve got an RRE Except question which we can identify from the question stem: Each of the following, if true, helps to resolve the survey’s apparently paradoxical results EXCEPT:

Let’s start by identifying the apparent paradox. The average number of books read annually per capita (i.e. per person) has gone down every year for three years. Bookstores, however, have increased their profits over that time period.

Any hypothetical resolutions? Well what if books now cost more? So people are reading less but the profit margins for bookstores are higher. What if a lot more people now live in the region we are surveying? Maybe the same number of people are reading the same (or more) books annually, but there are simply more non-readers in the population. This would mean the profits might not take a hit, but the average books read annually would decrease because we have added more non-readers to the pool. What if a bunch of bookstores closed over the past three years? So the amount of books that people are reading has gone down, but the business is now consolidated into a few bookstores who are now getting more business than before.

There are a ton of potential ways to reconcile this info. Which is great because this is an “except” question, meaning we are going to be given 4 answer choices that do reconcile the paradox, and only one that doesn’t. Our job is to find the latter.

Let’s move on to the answer choices:

Answer Choice (A) At first glance it is hard to see what this is doing for us. Libraries are purchasing fewer contemporary novels. Doesn’t that mean less books are being purchased, and isn’t that a fact that we already know about? Well, let’s take a step back. We know that fewer books are being read, but bookstores are making more money. This is giving us a reason why a certain segment of the population might be bringing new business to bookstores. It’s reasonable to assume that frequent library goers who prefer contemporary novels may now be forced to buy these books from bookstores where once they would have borrowed them from libraries. Therefore, even though our per capita rate of books read per year might go down, we have a fact that would lead to profits for bookstores increasing. This is reconciling our discrepancy, but remember, this is an except question–so this AC is wrong!

Correct Answer Choice (B) This is correct! It does absolutely nothing for our paradox! Shoplifting is affecting other businesses but not bookstores. That would give us a reason why profits for bookstores might be better compared to other businesses–but does it give us a reason why profits for bookstores would have gone up? We are told that the increase in shoplifting is recent. Therefore, it’s not like shoplifters were ravaging bookstores and shrinking their profit margins until bookstores got their act together recently and installed security systems. No! The scourge of shoplifting has only affected retail recently. Therefore, we don’t have anything here that would lead to an increase in profits. We just have a reason why profits might not be decreasing at the same rate as other businesses that are more affected by shoplifting. But that’s not a paradox that we care about. This is completely irrelevant to our stimulus, and because this question is an except question, it’s the right answer!

Answer Choice (C) This is great! It gives us an alternative revenue source that can account for the increased profits of bookstores, even if people are buying less books! Therefore, it is unfortunately incorrect since this question is an except question!

Answer Choice (D) This is telling us that people are spending more per book than they used to. This might mean that even if they are buying fewer books, bookstores are making higher profits. Since (D) helps resolve the discrepancy in the stimulus, (D) is wrong!

Answer Choice (E) Just like (C), this is giving us an alternative revenue source besides books. So people are reading fewer books, but they’re buying more magazines! Therefore, it’s quite possible that bookstores are making more money. It’s resolving the paradox in the stimulus, and therefore wrong.

 


47 comments

The question stem reads: The reasoning in which of the following is most similar to that in the naturalist's argument? This is a Parallel question.

The naturalist begins by claiming that a species can survive the change in an environment as long as the change is not too rapid. The naturalist has provided a general rule saying that the change can be ok for a species, with the caveat that the change does not occur too rapidly. The naturalist concludes that the threats humans create to woodland species arise not from cutting down trees but from the rate at which we are cutting down trees. The naturalist has applied the universal rule about species to the specific example of woodland species. So the problem is not that change we are creating by cutting down trees, but the because we are causing the change too rapidly.

When evaluating an answer choice, we need a universal rule with a caveat. The correct AC will apply that universal rule to a specific example and say that the specific example is failing to satisfy the caveat.

Answer Choice (A) is incorrect. (A) does not provide a universal rule; it only gives a specific rule about fossil fuels. Additionally, (A) 's rule about fossil fuels lacks the caveat we are looking for.

Answer Choice (B) is incorrect. We can quickly eliminate (B) because of the word "many." Remember, we need a universal rule, so if (B) was right, it would begin with "all people." Additionally, (B) 's rule lacks the caveat we are looking for, nor does (B) apply its rule to a specific example.

Answer Choice (C) is incorrect. Similar to (B), we can eliminate (C) because it says "some" when we are looking for a universal rule. Additionally, (C) also lacks the caveat, nor does (C) apply the rule to a specific example.

Correct Answer Choice (D) matches the stimulus. (D) provides a general rule that "people do not fear change," under the caveat people know what the change will bring. (D) then applies that rule to the specific example of the author's company's employees. The company's employees' fears arise from the fact the company is changing, but because they do not know what the change will bring (the caveat is not satisfied).

Answer Choice (E) is incorrect. (E) does not provide a general rule, so we can eliminate it.


23 comments